Un primo p t.c. p | (a^3 + 1), e però p non divide a + 1

Numeri interi, razionali, divisibilità, equazioni diofantee, ...
Rispondi
Avatar utente
HiTLeuLeR
Messaggi: 1874
Iscritto il: 01 gen 1970, 01:00
Località: Reggio di Calabria

Un primo p t.c. p | (a^3 + 1), e però p non divide a + 1

Messaggio da HiTLeuLeR »

Facile facile, ultimamente si vedono soltanto problemi astrusi, da queste parti. :roll:

Provare che, per ogni intero a > 2, esiste un primo naturale p t.c. p divide $ a^3 + 1 $, e tuttavia a + 1 non è divisibile per p.

NOTA: il problema (per quanto ne so) è di S. Leman.

EDIT: vai, Bollasso, che questo è tutto tuo! :wink:
Igor
Messaggi: 108
Iscritto il: 01 gen 1970, 01:00

Messaggio da Igor »

Abbiamo che $ a^3+1=(a+1)(a^2-a+1) $.

Ora calcoliamo il $ gcd $ tra $ a+1 $ e $ a^2-a+1 $:

$ (a+1,a^2-a+1)=(a+1,a^2-2a)=(a+1,a(a-2)) $

Ora,per $ a\ge 3 $ abbiamo che

$ (a+1,a(a-2))=(a+1,a-2)=1 $

Quindi,per $ a\ge 3 $ abbiamo che $ (a+1) $ e $ (a^2-a+1) $ sono coprimi.Allora

$ p|(a+1)(a^2-a+1)\Rightarrow p|(a+1)\wedge p|(a^2-a+1) $.

Allora se prendiamo un primo $ p $ che divide $ a^2-a+1 $ questo sicuramente non divide $ a+1 $ e questo conclude la dimostrazione.

P.S. Ciao HiTLeuLeR,da quanto tempo... :)
Avatar utente
HiTLeuLeR
Messaggi: 1874
Iscritto il: 01 gen 1970, 01:00
Località: Reggio di Calabria

Messaggio da HiTLeuLeR »

Igor ha scritto:[...] calcoliamo il $ gcd $ tra $ a+1 $ e $ a^2-a+1 $: $ (a+1,a^2-a+1)=(a+1,a^2-2a)=(a+1,a(a-2)) $. Ora, per $ a\ge 3 $ abbiamo che $ (a+1,a(a-2))=(a+1,a-2)=1 $

Quindi, per $ a\ge 3 $ abbiamo che $ (a+1) $ e $ (a^2-a+1) $ sono coprimi.
Mi spiace, ma questo non è vero! Se ad esempio $ a = 5 $, hai che $ \gcd(a+1,a^2 - a+1) = \gcd(6, 21) = 3 > 1 $. Qualcosa non torna, eh?! :lol: Vedi un po' di sistemarla, su... :roll:
Igor ha scritto:P.S. Ciao HiTLeuLeR,da quanto tempo... :)
Mmmh... Precisamente da quanto?! :shock: Comunque... ciao anche a te! :wink:
Igor
Messaggi: 108
Iscritto il: 01 gen 1970, 01:00

Messaggio da Igor »

Vediamo di rimediare:

Abbiamo che

$ (a+1,a^2-a+1)=(a+1,(a+1)^2-3a) $ $ =(-3a,a+1)=(3,a+1) $.

Allora o $ a+1 $ e $ a^2-a+1 $ sono coprimi,oppure il loro $ gcd $ è $ 3 $,il che avviene solo quando $ a\equiv 2\bmod 3 $.
Se sono coprimi la tesi è verificata per quanto detto nel post precedente,se invece il $ gcd $ è 3,consideriamo l'equazione

$ a^2-a+1=3^k $

Per $ k=0 $ abbiamo le soluzioni $ a=0 $ e $ a=1 $
Per $ k=1 $ abbiamo la soluzione $ a=2 $

Per $ k\ge 2 $ deve necessariamente essere

$ a^2-a+1\equiv 0\bmod 9 $

Poichè ci interessano solo gli $ a\equiv 2\bmod 3 $ possiamo limitarci a considerare $ a=2,5,8 $.

Per $ a=2 $ abbiamo $ 2^2-2+1\equiv 3\bmod 9 $
Per $ a=5 $ abbiamo $ 5^2-5+1\equiv 3\bmod 9 $
Per $ a=8 $ abbiamo $ 8^2-8+1\equiv 3\bmod 9 $

Quindi,per $ a\equiv 2\bmod 3 $ e $ a\ge 2 $,$ a^2-a+1 $ ha almeno un fattore primo $ p $ diverso da $ 3 $.Per quanto detto all'inizio allora, abbiamo che $ p $ non divide $ a+1 $ e quindi verifica la tesi.

Speriamo che ora vada meglio :oops:
HiTLeuLeR ha scritto:Precisamente da quanto?
Beh,ora non ti attaccare ai dettagli... :D .Comunque era un bel pò che non scrivevo sul forum.
Avatar utente
Poliwhirl
Messaggi: 383
Iscritto il: 01 gen 1970, 01:00
Località: Napoli

E dopo tanto tempo posto anch'io qualcosa...

Messaggio da Poliwhirl »

Problema: Provare che, per ogni intero $ a > 2 $, esiste un primo naturale $ p $ tale che $ p $ divide $ a^3 + 1 $, e tuttavia $ a + 1 $ non è divisibile per $ p $.

$ \displaystyle p\mid (a^3+1) \Longrightarrow p\mid (a+1)(a^2-a+1) $; supponiamo che $ \displaystyle p\mid (a^2-a+1) \wedge p\mid (a+1) $, quindi avremo che $ \displaystyle a^2-a\equiv -1 (\bmod p) \wedge a\equiv -1 (\bmod p) $ da cui $ \displaystyle a^2-a\equiv a (\bmod p) \Longrightarrow a^2-2a\equiv 0 (\bmod p) \Longrightarrow a(a-2)\equiv 0 (\bmod p) $; $ \displaystyle a\equiv 0 (\bmod p) $ è impossibile poiché $ \displaystyle a\equiv -1 (\bmod p) $; quindi $ \displaystyle a\equiv 2 (\bmod p) $, e poiché $ \displaystyle a\equiv -1 (\bmod p) $ allora $ \displaystyle 1\equiv -2 (\bmod p) \Longrightarrow 3\equiv 0 (\bmod p) \Longrightarrow p\mid 3 \Longrightarrow p=3 $; quindi abbiamo ottenuto che $ p $ divide sia $ \displaystyle a+1 $ sia $ \displaystyle a^2-a+1 $ solo quando è uguale a $ 3 $. Risulta che se $ \displaystyle a^2-a+1\neq 3k $ allora la tesi è dimostrata; se $ \displaystyle a^2-a+1 = 3k $ allora, dopo facili calcoli, si ottiene che $ \displaystyle a = \frac{1+\sqrt{3(k-1)}}{2} \wedge a = \frac{1-\sqrt{3(k-1)}}{2} $; poiché $ \displaystyle a>2 $ la seconda soluzione è da scartare; poiché $ a $ è intero allora necessariamente $ \displaystyle k = 3^s * t^2 +1 $ con $ \displaystyle s\in\mathbb{N^{*}} $ e $ \displaystyle t\in\mathbb{N} $, da cui $ \displaystyle 3\nmid k $; quindi $ k $ è prodotto di fattori primi diversi da $ 3 $ e che quindi non possono dividere $ \displaystyle a+1 $ c.v.d..

Bye,
#Poliwhirl#
Avatar utente
HiTLeuLeR
Messaggi: 1874
Iscritto il: 01 gen 1970, 01:00
Località: Reggio di Calabria

Messaggio da HiTLeuLeR »

Buono a entrambi! Vorrei farvi soltanto notare come tutti e due non avete utilizzato esplicitamente l'ipotesi secondo cui a > 2. E' vero sì, whirly, che tu ne hai fatto uso per escludere la determinazione positiva di una radice sbucata fuori nel corso delle tue argomentazioni, MA... non è lì che avresti dovuto ricorrerle! Tanto più che, se ci pensi un attimo, il problema continua comunque a valere supponendo (più in generale) $ a < -1 $ vel $ a > 2 $. Vi chiederete allora dov'è che l'ipotesi indicata interviene effettivamente nel corso della soluzione, no?! Beh, semplice: serve perché vi sia garantito $ a^2 - a +1 > 3 $. :wink:
Avatar utente
HiTLeuLeR
Messaggi: 1874
Iscritto il: 01 gen 1970, 01:00
Località: Reggio di Calabria

Messaggio da HiTLeuLeR »

Ecco la mia: se $ q \in \mathfrak{P} $ e $ q \mid \gcd(a+1, a^2 - a+1) $, allora $ a \equiv -1 \bmod q $, e perciò $ 0 \equiv a^2 - a + 1 \equiv 3 \bmod q $, i.e. $ q = 3 $. Inoltre, se pure $ 3 \mid \gcd(a+1,a^2 - a + 1) $, cosicché $ a = 3k-1 $ per qualche $ k\in\mathbb{Z} $, comunque $ a^2 - a + 1 $ $ \equiv (3k-1)^2 - (3k-1) + 1 $ $ \equiv -6k - 3k + 3 \equiv 3 \bmod 9 $, di modo che $ 9 \nmid (a^2 - a + 1) $. Ciò nondimeno $ a^2 - a + 1 > 3 $, se $ a > 2 $ oppure $ a < -1 $. Ne segue che, nei casi indicati, esiste almeno un primo naturale $ q $ tale che $ q \mid (a^3+1) $ e $ q \nmid (a+1) $, q.e.d.
Rispondi